SchneiderME01
Thanks Received: 0
Vinny Gambini
Vinny Gambini
 
Posts: 8
Joined: September 10th, 2011
 
 
 

Q22 - Radio airplay restrictions are

by SchneiderME01 Mon Jan 30, 2012 9:39 pm

So since this is a matching question I know my first step should be to simplify the arguement as much as possible which I think can be boiled down to something similar to...

Restrictions for R
X is ALL regulations except Y & Z
R's restrictions are ~Y and ~Z
So X's regulations cover R's restrictions

I feel like there must be a breakdown in how I broke the arguement down because I can't get the right answer.

I choose A which is the wrong answer

I think the answer choices break down as follows

A) All X pose a risk
Recommend avoid all X not high in Y
Many Z are low in Y
So Avoid Z

There seem to be too many variables a play here so it is not a good match

B) X is Y
Prize for each Y except S and ~N
X is N and ~S
So prize for X

This is the right answer but I'm not sure I see the fit.

The origional arguement didn't say X is Y but the rest of the arguement seems to fit because both arguements have an exception say that the origional didn't fit the exception so it must get the prize or be covered by the regulations

C) uses an example which isn't part of the origional arguement so I think we can rule it out on that alone but maybe I'm being to hasty with my eliminations

D) X is a kind of Y
Z does not produce Y that use A
Z don't use A
So Z produces X

The reasoning in this arguement appears to be flawed and the origional arguement isn't flawed so D wouldn't be the answer

E) T are a type of X
Association supports T that are Y even if not shown to reduce Z
Association will support ALL X that are Y

which isn't the same as the origional because you don't have the options or the exceptions.

Side question: on answer E even if isn't a conditional trigger if I'm remembering my class correctly even if is most of the time thrown in to be confusing but can usually be ignored

So I'm still confused about the right answer because my logic is flawed in that I could rule them all out
 
timmydoeslsat
Thanks Received: 887
Atticus Finch
Atticus Finch
 
Posts: 1136
Joined: June 20th, 2011
 
 
trophy
Most Thanked
trophy
First Responder
 

Re: Q22 - Radio airplay restrictions are

by timmydoeslsat Mon Jan 30, 2012 11:42 pm

I am VERY impressed with your breakdown of this question. You are doing a great job.

I agree with a lot of your analysis, particularly on choice D. That is an invalid argument, while the one in the stimulus is valid, which is reason enough to eliminate the answer choice.

So what is going on in this stimulus?

What we have here is is a group that compiled a guide of every regulation except those concerning tax or concerning labor.

Radio restrictions is not related to either of those two, so we can validly conclude that these restrictions are included in the guide.

So we want to mirror the idea of concluding that something is included in a group after determining that it is not among those that were eliminated.

That is the general idea of what happened in the stimulus. We can get more specific if we encounter multiple instances of this occurring in the answer choices. (For example, the stimulus introduces 2 ideas that the concluding item was not related to, maybe 2 choices remain in our list of answers, 1 having just one idea of narrowing, while the other had two.)

We can also determine any argument that is invalid as this would not be parallel at all.

A) This has a big wording change. It goes from talking about desserts to snack foods. Invalid.


B) So we know that C is a P.

Garden Club presents an award to each P except shrubs and those ~native to North America.

C is ~shrub and is native.

We can validly conclude that C will be presented an award.


C) We do not know if this Coalition poses a threat to regional economic growth! We cannot conclude anything about it validly.

This answer choice is assuming that the Coalition does not pose a threat to regional economic growth. Since this is not a stated premise, we have an invalid argument.


D) You got the analysis right on this one. Cannot conclude that the corporation produces the compact discs.


E) The association supports traffic laws that are in the public interest.

Yet this argument concludes that the association should support all government regulations that are in the public interest.

We are going from traffic laws in the PI to all government regulations in the PI. Big scope shift.

Your comment about even if is basically right.

Really, even if is just a way of showing that a conditional relationship is still intact.
User avatar
 
ManhattanPrepLSAT1
Thanks Received: 1909
Atticus Finch
Atticus Finch
 
Posts: 2851
Joined: October 07th, 2009
 
This post thanked 1 time.
 
 

Re: Q22 - Radio Airplay Restrictions

by ManhattanPrepLSAT1 Tue Feb 07, 2012 11:38 pm

Great discussion so far! I think you both have offered some really good ways to navigate this Match the Reasoning question. Let me add another perspective, though I don't mean for it to substitute the work so far, but rather offer another slightly different approach.

The stimulus offers a rule, and then outlines some exceptions. It then goes on to say that the exceptions are not applicable in the current situation, and so concludes that the rule can be applied.

If we look at the answer choices, the only two that offer exceptions to a general rule are answer choices (B) and (C). This allows us to eliminate answer choices (A), (D), and (E).

Timmy's point on answer choice (C) that there is an assumption is absolutely right. Although I see this as assuming that discouraging overdevelopment is not a threat to economic growth. If anything, discouraging development should be a threat to economic growth, and so I'm suspicious that the Neighborhood Association would be in favor of the Windsor Coalition. Regardless, this is a flawed argument and so does not match the reasoning in the stimulus.

Leaving us with answer choice (B). There's a rule, some exceptions to the rule, a statement that says the exceptions do not apply, and then a conclusion that states that the rule is therefore applicable - just as in the stimulus.

Nice work timmydoeslsat and SchneiderME01!
 
donghai819
Thanks Received: 7
Elle Woods
Elle Woods
 
Posts: 65
Joined: September 25th, 2015
 
 
 

Re: Q22 - Radio airplay restrictions are

by donghai819 Fri Jan 08, 2016 3:49 pm

The reasoning here is similar to "reach the conclusion by ruling out a potential, contradictory alternative explanation" in procedure questions.
 
obobob
Thanks Received: 1
Elle Woods
Elle Woods
 
Posts: 78
Joined: January 21st, 2018
 
 
 

Re: Q22 - Radio airplay restrictions are

by obobob Sun Apr 28, 2019 12:58 am

Hi, Can anyone please explain what's wrong with (D)'s argument?
Is it an invalid argument that assumes that Leotol Corporation produces compact discs just because discs possess certain quality (data storage) that is also possessed by the corporation's products?
User avatar
 
ohthatpatrick
Thanks Received: 3808
Atticus Finch
Atticus Finch
 
Posts: 4661
Joined: April 01st, 2011
 
 
 

Re: Q22 - Radio airplay restrictions are

by ohthatpatrick Tue Apr 30, 2019 12:14 am

Yeah, one way to get rid of it is because (A) was an airtight argument, where we HAD to believe the conclusion. (D) is not.

They proved to us that radio restrictions will be in the CC guide, because
radio restrictions = nationally imposed regulations
CC guide = contains all nationally imposed regulations
(other than a couple exceptional cases, neither of which apply to radio restrictions)

In (D), they did not prove that LC makes cd's.
All we know about LC is that it doesn't produce analog storage devices.
Cool. That doesn't mean that it DOES produce digital storage devices (like cd's).

When you're looking at the original conclusion, it's about the CC guide.
What do we know about the CC guide?
It covers ALL national regulations, other than these two exceptional cases.

For (D) to be more promising, since it's conclusion is about LC,
we should have a premise that says,
LC does ALL _____ , other than this exceptional case.

Instead, we only hear about the exceptional case (it does not do analog storage devices).

We would have needed to hear,
LC produces all types of storage devices, except those that use analog storage methods.

If THAT had been the 2nd sentence, the argument would be sound and would match the original.